Pagina 1 di 1

Tournament of the Towns 1987

Inviato: 09 nov 2008, 13:18
da mod_2
Dimostrare che la disuguaglianza $ $\sqrt{2\sqrt{3\sqrt{4 \cdots \sqrt{(n-1)\sqrt{n}}}}}<3$ $ è valida per qualunque n naturale.
Tournament of the Towns 1987

E' veramente facile e quindi gli esperti sono pregati di non bruciarlo in 10 secondi :D

Inviato: 09 nov 2008, 15:36
da Davide90
Dimostriamolo per induzione.
Innanzitutto la disuguaglianza è vera (partiamo da un caso non banale) per $ $ n = 3 $, infatti $ \displaystile \sqrt {2\sqrt{3}} <3 \Leftrightarrow 2\sqrt{3} <9 $ , ma $ \displaystile 2\sqrt{3} <2\cdot 3 < 9 $ .
Ora, supponiamo la disuguaglianza vera per $ $ n $ : allora $ \displaystile $\sqrt{2\sqrt{3\sqrt{4 \cdots \sqrt{(n-1)\sqrt{n}}}}} = (2^{2^{n-1}}\cdot 3^{2^{n-2}} \cdot \dots \cdot (n-1)^2 \cdot n )^{\frac{1}{2^n} } <3 $ . Indichiamo con $ \mathbb{P} $ il LHS della disuguaglianza.
Dimostrare che la disuguaglianza vale anche per $ n+1 $ equivale a dimostrare che $ \displaystile \sqrt {\mathbb{P} \cdot (n+1)^{\frac {1}{2^n}}} <3 $ . elevando al quadrato si ottiene
$ \displaystile \mathbb{P} \cdot (n+1)^{\frac {1}{2^n}} <9 $ , ma $ \displaystile \mathbb{P} < 3 $ , perciò si deve dimostrare che
$ \displaystile (n+1)^{\frac {1}{2^n}}<3 \Leftrightarrow n +1 < 3 ^{2^n}} $ , vero $ \forall n \in \mathbb {N} $ .
Spero di non sbagliarmi... :oops:

Inviato: 09 nov 2008, 15:41
da julio14
Il P è fuori dalla radice

Inviato: 09 nov 2008, 15:49
da Davide90
Già, hai ragione... perciò dovrei dimostrare che $ \displaystile\mathbb{P} \cdot (n+1)^{\frac {1}{2^{n+1}}} <3 $ .... :roll:

Inviato: 09 nov 2008, 16:14
da SkZ
in verita' andava tolta la radice. E'
$ \displaystile\mathbb{P} \cdot (n+1)^{\frac {1}{2^{n}}} <3 $
ogni numero e' sotto n-1 radici

Inviato: 09 nov 2008, 16:27
da kn
Davide90 ha scritto:$ \displaystile $\sqrt{2\sqrt{3\sqrt{4 \cdots \sqrt{(n-1)\sqrt{n}}}}} = (2^{2^{n-1}}\cdot 3^{2^{n-2}} \cdot \dots \cdot (n-1)^2 \cdot n )^{\frac{1}{2^n} } <3 $
Sono d'accordo con SkZ, qui dovevi scrivere
$ \displaystile $\sqrt{2\sqrt{3\sqrt{4 \cdots \sqrt{(n-1)\sqrt{n}}}}} = (2^{2^{n-2}}\cdot 3^{2^{n-3}} \cdot \dots \cdot (n-1)^2 \cdot n )^{\frac{1}{2^{n-1}} } <3 $ :wink:

Inviato: 09 nov 2008, 20:27
da pak-man
C'è qualcosa che non mi convince: nel passo induttivo bisogna dimostrare che $ \mathbb{P}\sqrt[2^n]{n+1}<3 $, e poiché l'ipotesi dell'induzione è $ \mathbb{P}<3 $ dividendo dobbiamo dimostrare che $ \sqrt[2^n]{n+1}<1 $, che è chiaramente impossibile!:x Dove ho sbagliato? :? :(

tentativo...

Inviato: 09 nov 2008, 20:34
da Jack Luminous
Io pensavo di usare l'induzione in una maniera più 'forte', visto che quella standard fallisce come detto sopra... dite che è completamente insensato cercare di mostrare che
$ \displaystyle\sqrt{2\sqrt{3\sqrt{4 \cdots \sqrt{(n-1)\sqrt{n}}}}}<3-\frac1n $
?

Re: tentativo...

Inviato: 09 nov 2008, 20:54
da pak-man
Jack Luminous ha scritto:Io pensavo di usare l'induzione in una maniera più 'forte', visto che quella standard fallisce come detto sopra... dite che è completamente insensato cercare di mostrare che
$ \displaystyle\sqrt{2\sqrt{3\sqrt{4 \cdots \sqrt{(n-1)\sqrt{n}}}}}<3-\frac1n $ ?
mmm...non mi convince molto (ma potrei anche sbagliarmi, ovviamente):
$ p\sqrt[2^n]{n+1}<3+\frac{1}{n} $
$ \sqrt[2^n]{n+1}<1+\frac{1}{3n} $
e non vedo come si possa andare avanti...

Inviato: 09 nov 2008, 21:09
da Jack Luminous
peccato :oops:

Inviato: 09 nov 2008, 21:18
da SkZ
il fatto e' che 5<20, anche 5*3<20, ma 3>1

sembra tendere a 2.76...
quindi quella limitazione dovrebbe andar bene

Inviato: 10 nov 2008, 21:43
da mod_2
qualcuno vuole provare con l'induzione a scendere? :D

Inviato: 13 nov 2008, 20:56
da atat1tata
mod_2 ha scritto:qualcuno vuole provare con l'induzione a scendere? :D
Chiedo venia, che significa induzione a scendere?

Credo (spero) di aver fatto una sottospecie obbrobriosa di dimostrazione con logaritmi a manetta e un doppio ricorso all'induzione. Mi vergogno di postarla qui :oops:

Inviato: 14 nov 2008, 19:53
da mod_2
atat1tata ha scritto:
mod_2 ha scritto:qualcuno vuole provare con l'induzione a scendere? :D
Chiedo venia, che significa induzione a scendere?

Credo (spero) di aver fatto una sottospecie obbrobriosa di dimostrazione con logaritmi a manetta e un doppio ricorso all'induzione. Mi vergogno di postarla qui :oops:
ma, niente di speciale, forse era meglio tradurlo con "l'induzione inversa".
Generalizziamo e mettiamo m al posto di 2, m+1 al posto di 3 e così via, si dimostra innanzitutto che la disuguaglianza è valida nel caso m=n, e poi per tutti gli m<n.
Se hai una soluzione diversa, postala comunque, anche se per te è brutta. Sono curioso di vederla :D